K
Khách

Hãy nhập câu hỏi của bạn vào đây, nếu là tài khoản VIP, bạn sẽ được ưu tiên trả lời.

16 tháng 8 2019

\(\frac{1}{x^4}+\frac{1}{y^4}=\frac{x^2}{x^6}+\frac{1}{y^4}\ge\frac{\left(x+1\right)^2}{x^6+y^4}\ge\frac{4x}{x^6+y^4}\)

tương tự

\(\frac{1}{y^4}+\frac{1}{z^4}\ge\frac{4y}{y^6+z^4}\);

\(\frac{1}{z^4}+\frac{1}{x^4}\ge\frac{4z}{z^6+x^4}\);

cộng vế với vế => đpcm

Dấu "=" xảy ra <=> x=y=z=1

16 tháng 8 2019

Cô si

1 tháng 7 2015

Áp dụng bất đẳng thức Cauchy - Schwarz : \(\frac{a^2}{b}+\frac{c^2}{d}\ge\frac{\left(a+c\right)^2}{b+d}\) 

\(\frac{1}{x^4}+\frac{1}{y^4}=\frac{x^2}{x^6}+\frac{1^2}{y^4}\ge\frac{\left(x+1\right)^2}{x^6+y^4}\ge\frac{4x}{x^6+y^4}\)(\(\left(a+b\right)^2\ge4a\))

Tương tự: \(\frac{1}{y^4}+\frac{1}{z^4}\ge\frac{4y}{y^6+z^4};\frac{1}{z^4}+\frac{1}{x^4}\ge\frac{4z}{z^6+x^4}\)

\(\Rightarrow2.\left(\frac{1}{x^4}+\frac{1}{y^4}+\frac{1}{z^4}\right)\ge4\left(\frac{x}{x^6+y^4}+\frac{y}{y^6+z^4}+\frac{z}{z^6+x^4}\right)\)

\(\Rightarrow\frac{1}{x^4}+\frac{1}{y^4}+\frac{1}{z^4}\ge\frac{2x}{x^6+y^4}+\frac{2y}{y^6+z^4}+\frac{2z}{z^6+x^4}\)

Dấu "=" xảy ra khi và chỉ khi \(x=y=z=1\)

 

22 tháng 6 2018

với x,y,z >0 áp dụng bđt cosi ta có:

\(x^6+y^4>=2\sqrt{x^6y^4}=2x^3y^2\Rightarrow\frac{2x}{x^6+y^4}< =\frac{2x}{2x^3y^2}=\frac{1}{x^2y^2}\)

\(y^6+z^4>=2\sqrt{y^6z^4}=2y^3z^2\Rightarrow\frac{2y}{y^6+z^4}< =\frac{2y}{2y^3z^2}=\frac{1}{y^2z^2}\)

\(z^6+x^4>=2\sqrt{z^6x^4}=2z^3x^2\Rightarrow\frac{2z}{z^6+x^4}< =\frac{2z}{2z^3x^2}=\frac{1}{z^2x^2}\)

\(\Rightarrow\frac{2x}{x^6+y^4}+\frac{2y}{y^6+z^4}+\frac{2z}{z^6+x^4}< =\frac{1}{x^2y^2}+\frac{1}{y^2z^2}+\frac{1}{z^2x^2}\left(1\right)\)

với x,y,z>0 áp dụng bđt cosi ta có:

\(\frac{1}{x^4}+\frac{1}{y^4}>=2\sqrt{\frac{1}{x^4}\cdot\frac{1}{y^4}}=\frac{2}{x^2y^2}\)

\(\frac{1}{y^4}+\frac{1}{z^4}>=2\sqrt{\frac{1}{y^4}\cdot\frac{1}{z^4}}=\frac{2}{y^2z^2}\)

\(\frac{1}{x^4}+\frac{1}{z^4}>=2\sqrt{\frac{1}{x^4}\cdot\frac{1}{z^4}}=\frac{2}{x^2z^2}\)

\(\Rightarrow\frac{2}{x^4}+\frac{2}{y^4}+\frac{2}{z^4}>=\frac{2}{x^2y^2}+\frac{2}{y^2z^2}+\frac{2}{x^2z^2}\Rightarrow\frac{1}{x^4}+\frac{1}{y^4}+\frac{1}{z^4}>=\frac{1}{x^2y^2}+\frac{1}{y^2z^2}+\frac{1}{x^2z^2}\)

\(\Rightarrow\frac{1}{x^2y^2}+\frac{1}{y^2z^2}+\frac{1}{x^2z^2}< =\frac{1}{x^4}+\frac{1}{y^4}+\frac{1}{z^4}\left(2\right)\)

từ \(\left(1\right)\left(2\right)\Rightarrow\frac{2x}{x^6+y^4}+\frac{2x}{y^6+z^4}+\frac{2x}{z^6+x^4}< =\frac{1}{x^4}+\frac{1}{y^4}+\frac{1}{z^4}\)(đpcm)

dấu = xảy ra khi x=y=z=1

14 tháng 5 2017

bạn xem câu hỏi số 905663 nhé

8 tháng 5 2017

Đề kì vậy bạn. Sao vế trái không có \(y\) vậy?

11 tháng 5 2017

Bài 2: 

\(a^4+b^4\ge a^3b+b^3a\)

\(\Leftrightarrow a^4-a^3b+b^4-b^3a\ge0\)

\(\Leftrightarrow a^3\left(a-b\right)-b^3\left(a-b\right)\ge0\)

\(\Leftrightarrow\left(a-b\right)^2\left(a^2+ab+b^2\right)\ge0\)

ta thấy : \(\orbr{\orbr{\begin{cases}\left(a-b\right)^2\ge0\\\left(a^2+ab+b^2\right)\ge0\end{cases}}}\Leftrightarrow dpcm\)

Dấu " = " xảy ra khi a = b

tk nka !!!! mk cố giải mấy bài nữa !11

27 tháng 3 2019

1/Thêm 6 vào 2 vế,ta cần c/m:

\(\left(x^4+1+1+1\right)+\left(y^4+1+1+1\right)\ge8\)

Thật vậy,áp dụng BĐT AM-GM cho cái biểu thức trong ngoặc,ta được:

\(VT\ge4\left(x+y\right)=4.2=8\) (đpcm)

Dấu "=" xảy ra khi x = y = 1 (loại x = y = -1 vì không thỏa mãn x + y = 2)

AH
Akai Haruma
Giáo viên
17 tháng 7 2019

Bài 1:

Áp dụng BĐT Bunhiacopxky:

\((a^2+b^2+c^2+d^2)(1+1+1+1)\geq (a+b+c+d)^2\)

\(\Leftrightarrow a^2+b^2+c^2+d^2\geq \frac{(a+b+c+d)^2}{4}=\frac{2^2}{4}=1\) (đpcm)

Dấu "=" xay ra khi \(a=b=c=d=\frac{1}{2}\)

AH
Akai Haruma
Giáo viên
17 tháng 7 2019

Bài 2:

Bạn xem lại đề:

Áp dụng BĐT Cô-si cho các số không âm ta có:

\(16a^4+1\geq 2\sqrt{16a^4.1}=8a^2\Rightarrow \frac{a^2}{1+16a^4}\leq \frac{a^2}{8a^2}=\frac{1}{8}(1)\)

\(b^4+1\geq 2\sqrt{b^4.1}=2b^2\Rightarrow \frac{b^2}{1+b^4}\leq \frac{b^2}{2b^2}=\frac{1}{2}(2)\)

Từ \((1);(2)\Rightarrow \frac{a^2}{1+16a^4}+\frac{b^2}{1+b^4}\leq \frac{1}{8}+\frac{1}{2}=\frac{5}{8}\) chứ không phải $\frac{1}{4}$

Nếu bạn muốn kết quả là $\frac{1}{4}$ thì cần thay $b^4$ bằng $16b^4$ và làm tương tự như trên.

1) Cho x > 1. Tìm GTNN của:   ​\(A=\frac{1+x^4}{x\left(x-1\right)\left(x+1\right)}\)2) Trong các cặp (x;y) thỏa mãn \(\frac{x^2-x+y^2-y}{x^2+y^2-1}\le0\). Tìm cặp có tổng x + 2y lớn nhất.3) Cho x thỏa mãn \(x^2+\left(3-x\right)^2\ge5\). Tìm GTNN của \(A=x^4+\left(3-x\right)^4+6x^2\left(3-x\right)^2\)4) Tìm GTNN của \(Q=\frac{1}{2}\left(\frac{x^{10}}{y^2}+\frac{y^{10}}{x^2}\right)+\frac{1}{4}\left(x^{16}+y^{16}\right)-\left(1+x^2y^2\right)^2\)5) Cho x, y > 1. Tìm...
Đọc tiếp

1) Cho x > 1. Tìm GTNN của:   ​\(A=\frac{1+x^4}{x\left(x-1\right)\left(x+1\right)}\)

2) Trong các cặp (x;y) thỏa mãn \(\frac{x^2-x+y^2-y}{x^2+y^2-1}\le0\). Tìm cặp có tổng x + 2y lớn nhất.

3) Cho x thỏa mãn \(x^2+\left(3-x\right)^2\ge5\). Tìm GTNN của \(A=x^4+\left(3-x\right)^4+6x^2\left(3-x\right)^2\)

4) Tìm GTNN của \(Q=\frac{1}{2}\left(\frac{x^{10}}{y^2}+\frac{y^{10}}{x^2}\right)+\frac{1}{4}\left(x^{16}+y^{16}\right)-\left(1+x^2y^2\right)^2\)

5) Cho x, y > 1. Tìm GTNN của \(P=\frac{\left(x^3+y^3\right)-\left(x^2+y^2\right)}{\left(x-1\right)\left(y-1\right)}\)

6) Cho x, y, z > 0 thỏa mãn: \(xy^2z^2+x^2z+y=3z^2\). Tìm GTLN của \(P=\frac{z^4}{1+z^4\left(x^4+y^4\right)}\)

7) Cho a, b, c > 0. CMR:\(\frac{a^2}{b^2+c^2}+\frac{b^2}{a^2+c^2}+\frac{c^2}{a^2+b^2}\ge\frac{a}{b+c}+\frac{b}{a+c}+\frac{c}{a+b}\)

8) Cho x>y>0. và \(x^5+y^5=x-y\). CMR: \(x^4+y^4<1\)

9) Cho \(1\le a,b,c\le2\). CMR: \(\left(a+b+c\right)\left(\frac{1}{a}+\frac{1}{b}+\frac{1}{c}\right)\le10\)

10) Cho \(x,y,z\ge0\)CMR: \(\sqrt[3]{x}+\sqrt[3]{y}+\sqrt[3]{z}\le\sqrt[3]{\frac{x+y}{2}}+\sqrt[3]{\frac{y+z}{2}}+\sqrt[3]{\frac{z+x}{2}}\)

11) Cho \(x,y\ge0\)thỏa mãn \(x^2+y^2=1\)CMR: \(\frac{1}{\sqrt{2}}\le x^3+y^3\le1\)

12) Cho a,b,c > 0 và a + b + c = 12. CM: \(\sqrt{3a+2\sqrt{a}+1}+\sqrt{3b+2\sqrt{b}+1}+\sqrt{3c+2\sqrt{c}+1}\le3\sqrt{17}\)

13) Cho x,y,z < 0 thỏa mãn \(x+y+z\le\frac{3}{2}\). CMR: \(\sqrt{x^2+\frac{1}{x^2}}+\sqrt{y^2+\frac{1}{y^2}}+\sqrt{z^2+\frac{1}{z^2}}\ge3\sqrt{17}\)

14) Cho a,b > 0. CMR: \(\left(\sqrt[6]{a}+\sqrt[6]{b}\right)\left(\sqrt[3]{a}+\sqrt[3]{b}\right)\left(\sqrt{a}+\sqrt{b}\right)\le4\left(a+b\right)\)

15) Với a, b, c > 0. CMR: \(\frac{a^8+b^8+c^8}{a^3.b^3.c^3}\ge\frac{1}{a}+\frac{1}{b}+\frac{1}{c}\)

16) Cho x, y, z > 0 và \(x^3+y^3+z^3=1\)CMR: \(\frac{x^2}{\sqrt{1-x^2}}+\frac{y^2}{\sqrt{1-y^2}}+\frac{z^2}{\sqrt{1-z^2}}\ge2\)

3
20 tháng 1 2016

cậu đăng mỗi lần 1 đến 2 câu thôi chứ nhiều thế này ai làm cho hết được

20 tháng 1 2016

Ok lần đầu mình đăng nên chưa biết, cảm ơn cậu đã góp ý, mình sẽ rút kinh nghiệm!!